Solve the given inequality. Round to the nearest ten-thousandth, if necessary. e x > 14

Answers

Answer 1

Answer:

[tex]x\in(1.146,\infty)[/tex]

Step-by-step explanation:

We are given an inequality

[tex]e^x>14[/tex]

We have to solve the given inequality.

Taking both side ln of given inequality

Then, we get

[tex]ln(e^x)>ln(14)[/tex]

[tex]xlne>1.146[/tex]

We know that

lne=1

Using the value

[tex]x>1.146[/tex]

[tex]x\in(1.146,\infty)[/tex]

Hence, the value of x is given by

[tex]x\in(1.146,\infty)[/tex]


Related Questions

f(x)=2x-6 g(x)=3x+9, find (f+g)

Answers

Answer:

  (f+g)(x) = 5x +3

Step-by-step explanation:

(f+g)(x) = f(x) +g(x)

  = (2x -6) +(3x +9)

  = 2x +3x -6 +9

(f+g)(x) = 5x +3

A section of an exam contains two multiple-choice questions, each with three answer choices (listed "A", "B", and "C"). Assuming the outcomes to be equally likely, find the probability (as a reduced fraction) that neither of the answers is "B". [Hint: List all the outcomes of the sample space first.]

Answers

Answer:

[tex]p = \frac{4}{9}[/tex]

Step-by-step explanation:

A probability is the number of desired outcomes divided by the number of total outcomes.

Sample space:

All posible outcomes.

First answer - Second answer:

A - A

A - B

A - C

B - A

B - B

B - C

C - A

C - B

C - C

9 possible outcomes.

Probability that neither is B.

A - A

A - C

C - A

C - C

4 possible outcomes.

So

[tex]p = \frac{4}{9}[/tex]

Two models R1 and R2 are given for revenue (in billions of dollars per year) for a large corporation. The model R1 gives projected annual revenues from 2008 through 2015, with t = 8 corresponding to 2008, and R2 gives projected revenues if there is a decrease in the rate of growth of corporate sales over the period. Approximate the total reduction in revenue if corporate sales are actually closer to the model R2. (Round your answer to three decimal places.) R1 = 7.21 + 0.55t R2 = 7.21 + 0.44t

Answers

Answer:

7.0422  is the correct answer to the given question .

Step-by-step explanation:

Given

R1 = 7.21 + 0.55t

R2 = 7.21 + 0.44t

Decrease in the revenue  can be determined by the formula

[tex]= Reduction\ in\ R1\ -\ Reduction\ in\ R2[/tex]

[tex]= (7.21 + 0.55t ) - (7.21 + 0.44t)[/tex]

=0.11 t

Now overall Reduction can be determined by the interval from t=8 to t=15

Consider c=0.11 t

[tex]\frac{dc}{dt}[/tex]=0.11

Now integrated the equation from t=8 to t=15 to determine total reduction in revenue

[tex]=\int_{8}^{15}\sqrt{1+0.11^2}\ dL[/tex]

[tex]=7.0422[/tex]

It costs Neil $2.88 to make two dozen muffins. He sold the muffins for $.80 each. If Neil sold six dozen muffins how much profit did he make from the sale?

Answers

Hi!

So for this you need an equation. It will be 72 • 0.8 - 2.88 • 3

this is bc 12 x 6 is 72 and 3 bc 2 x 3 is 6.

so after you solve it you should get : $48.96..Hope this helped! :))

How do i solve 3/4 x 4 7/12 =

Answers

Answer:

2.9375

Step-by-step explanation:

3/4×47/12

3×47/4×12

141/48

2.9375

Answer:

4x12=48

48+7=55

4 7/12 as an improper fraction is 55/12

multiply the numerator with numerator and denominator with denominator

3x55=165

4x12=48

165/48 can be divided by 3

165/3=55__48/3=16

55/16 is simplified

55/16=3 r 7 so the fraction is 3 7/16

Hope this helps

Step-by-step explanation:

If 25 new born babies are randomly selected, what is the probability that the 25 babies are born that their mean weigh less than 3100g?

Answers

Answer:

Step-by-step explanation:

Given that

n = 25

mean = 3100

standard deviation = 500

[tex]\bar x = \frac{500}{\sqrt{25} }[/tex]

= 500 / 5

= 100

N ( μ = 3500, (500)² / 25)

N = (3500,(100)² )

b) P ( x < 3100)

c) Z score

[tex]z = \frac{x - u _{\bar x}}{\sigma \_ {\bar x}}[/tex]

[tex]u_{\bar x}= 3500[/tex]

[tex]\sigma _{\bar x}= 100[/tex]

[tex]z = \frac{3100-3500}{100} \\\\= -4[/tex]

d)

probability=

P ( x < 3100) = P < -4 = 0

Answer:

a) The parameters for the sampling distribution include

Mean = μₓ = 3500 g

Standard deviation = σₓ = 100 g

Required probability = P(x < 3100)

b) The image of the probability density curve is presented in the attached image.

c) The z-score for 3100 g in the sampling distribution = -4.00

d) The probability that the 25 babies that are born that their mean weigh less than 3100 g = 0.00003

Step-by-step explanation:

Complete Question

Suppose we know the birth weights of babies is normally distributed, with mean of 3500 g and standard deviation 500g.

If 25 new born babies are randomly selected, what is the probability that the 25 babies are born that their mean weigh less than 3100g?

a) What are the parameters?

b) Draw a probability density curve for the problem.

c) What is the z-score of the weight?

d) What is the required probability?

Solution

The Central limit theorem explains that for a random sample of adequate size obtained from a normal distribution with independent variables, the mean of the sampling distribution is approximately equal to the population mean and the sampling distribution is approximately of the nature of the population distribution (normal) with the standard deviation of the sampling distribution given as

Standard deviation of the sampling distribution = [(Population Standard deviation)/√n]

σₓ = (σ/√n)

Mean of Sampling distribution = Population mean

μₓ = μ = 3500 g

σₓ = (σ/√n) = (500/√25) = 100 g

a) The parameters for the sampling distribution include

Mean = μₓ = 3500 g

Standard deviation = σₓ = 100 g

Required probability = P(x < 3100)

b) The image of the probability density curve is presented in the attached image.

c and d) To obtain the required probability, we need the z-score for 3100 g, so we standardize 3100g

The standardized score for any value is the value minus the mean then divided by the standard deviation.

z = (x - μ)/σ = (3100 - 3500)/100 = - 4.00

To determine the required probability 45mg/L, P(x < 3100) = P(z < -4.00)

We'll use data from the normal probability table for these probabilities

P(x < 3100) = P(z < -4.00) = 0.00003

Hence, the probability that the 25 babies that are born that their mean weigh less than 3100 g = 0.00003

Hope this Helps!!!!

Lines sand tare perpendicular. If the slope of line sis 5, what is the slope of
line t?

Answers

Answer:

  -1/5

Step-by-step explanation:

Perpendicular lines have slopes that are the opposite reciprocal of one another.

The slope of line t is the opposite reciprocal of the slope of line s:

  slope of t = -1/(slope of s)

  slope of t = -1/5

The accompanying data set consists of observations on shower-flow rate (L/min) for a sample of n = 129 houses: 4.6 12.1 7.1 7.0 4.0 9.2 6.7 6.9 11.5 5.1 11.2 10.5 14.3 8.0 8.8 6.4 5.1 5.6 9.6 7.5 7.5 6.2 5.8 2.7 3.4 10.4 9.8 6.6 3.7 6.4 8.3 6.5 7.6 9.3 9.2 7.3 5.0 6.3 13.2 6.2 5.4 4.8 7.5 6.0 6.9 10.8 7.5 6.6 5.0 3.3 7.6 3.9 11.9 2.2 15.0 7.2 6.1 15.3 18.1 7.2 5.4 5.5 4.3 9.0 12.7 11.3 7.4 5.0 3.5 8.2 8.4 7.3 10.3 11.9 6.0 5.6 9.5 9.3 10.4 9.7 5.1 6.7 10.2 6.2 8.4 7.0 4.8 5.6 10.5 14.6 10.8 15.5 7.5 6.4 3.4 5.5 6.6 5.9 15.0 9.6 7.8 7.0 6.9 4.1 3.6 11.9 3.7 5.7 6.8 11.3 9.3 9.6 10.4 9.3 6.9 9.8 9.1 10.6 4.5 6.2 8.3 3.1 4.9 5.0 6.0 8.2 6.3 3.8 6.0 (a) Construct a stem-and-leaf display of the data. (Enter numbers from smallest to largest separated by spaces. Enter NONE for stems with no values.)

Answers

Answer:

Step-by-step explanation:

For n=129 and with leaf unit = 0.1, the stem and leaf chart of the given data on Shower-flow rate (L/min) is as follows:

2 28

Stem        leaves

3----------1344567789

4----------01356889

5----------00001114455666789

6----------0000122223344456667789999

7----------00012233455555668

8----------02233448

9----------012233335666788

10----------2344455688

11--------- 2335999

12---------- 17

13-------- 9

14--------36

15---------- 0035

16---------None

17---------None

18 ----------3

* From steam and leaf chart we note that minimum Shower flow rate is 2.2 whereas maximum is 18.3 L/mim. Further typical or representative rate is 7.0 L/min.

* The display of data on steam and leaf chart shows that data is positively skewed means concentration of data on left side or lower value side is high as compared to other side.

* Distribution is not symmetric rather very clear positive skew ness is observed through steam and leaf chart. Even distribution is Unimodal.

* From steam and leaf chart is indicative to conclude that the highest observation 18.3 is outlier.

81. Restaurants. About 12% of the restaurants in the US are pizzerias, and there are about 70,000 pizzerias in the US. Estimate the total number of restaurants in the United States.

Answers

Answer:

For this case we know that 12% of the restaurants in the Us are pizzerias and there are about 70000 pizzerias in Us. So then we can apply a proportion rule given by:

[tex]\frac{70000}{12} = \frac{x}{100}[/tex]

Where x represent the total of restaurants representing the 100% and if we solve for x we got:

[tex] x = 100 \frac{70000}{12}= 583333.33[/tex]

So then there is approximately between 58333 and 58334 restaurants in US with the result obtained.

Step-by-step explanation:

For this case we know that 12% of the restaurants in the Us are pizzerias and there are about 70000 pizzerias in Us. So then we can apply a proportion rule given by:

[tex]\frac{70000}{12} = \frac{x}{100}[/tex]

Where x represent the total of restaurants representing the 100% and if we solve for x we got:

[tex] x = 100 \frac{70000}{12}= 583333.33[/tex]

So then there is approximately between 58333 and 58334 restaurants in US with the result obtained.

A cellular phone network uses towers to transit calls. Each tower transmits a circular area. on a grid of a city, the equations given represent the transmission boundaries of the towers. Tell which towers, is any, transmit to a phone located at M(3.5, 4.5).


Tower A: x^2 + y^2 = 9

Tower B: (x - 5)^2 + (y - 3)^2 = 6.25

Tower C: (x - 2)^2 + (y - 5)^2 = 4

Answers

Answer:

B

Step-by-step explanation:

Notice that

[tex](3.5-5)^2 + (4.5-3)^2 = 4.5[/tex]

Since 4.5 is less than 6.25, tower B transmits to that phone

Answer:

Towers B and C transmit to the phone.

Step-by-step explanation:

First, let see the location of the center for each tower:

Tower A

[tex]A (x,y) = (0,0)[/tex]

Tower B

[tex]B (x,y) = (5,3)[/tex]

Tower C

[tex]C(x,y) = (2,5)[/tex]

Now, the distance between the location of the phone and any of the towers by means of the Pythagorean equation. The phone is under the influence of a tower only if distance is less than transmission boundaries. Then:

Tower A

[tex]d_{A} = \sqrt{(3.5-0)^{2}+(4.5-0)^{2}}[/tex]

[tex]d_{A} \approx 5.701[/tex]

[tex]d_{A} > 3[/tex]

Tower B

[tex]d_{B} = \sqrt{(3.5-5)^{2}+(4.5-3)^{2}}[/tex]

[tex]d_{B} \approx 2.121[/tex]

[tex]d_{B} < 2.5[/tex]

Tower C

[tex]d_{C} = \sqrt{(3.5-2)^{2}+(4.5-5)^{2}}[/tex]

[tex]d_{C} \approx 1.581[/tex]

[tex]d_{C} < 2[/tex]

Towers B and C transmit to the phone.

Convert the line integral to an ordinary integral with respect to the parameter and evaluate it. ModifyingBelow Integral from nothing to nothing With Upper C (y minus z )ds∫C(y−z)ds​; C is the helix left angle 12 cosine t comma 12 sine t comma t right angle12cost,12sint,t​, for 0 less than or equals t less than or equals 2 pi0≤t≤2π The value of the ordinary integral is nothing.

Answers

C is parameterized by

[tex]\vec r(t)=\langle x(t),y(t),z(t)\rangle=\left\langle12\cos t,12\sin t,t\right\rangle[/tex]

for 0 ≤ t ≤ 2π. In the integral, replace y and z as above, and the line element ds is

[tex]\mathrm ds=\sqrt{\left(\dfrac{\mathrm dx}{\mathrm dt}\right)^2+\left(\dfrac{\mathrm dy}{\mathrm dt}\right)^2+\left(\dfrac{\mathrm dz}{\mathrm dt}\right)^2}\,\mathrm dt=\sqrt{145}\,\mathrm dt[/tex]

So the integral is

[tex]\displaystyle\int_C(y-z)\,\mathrm ds=\sqrt{145}\int_0^{2\pi}(12\sin t-t)\,\mathrm dt[/tex]

sin(t) has period 2π, so that term contributes nothing to the integral, leaving us with

[tex]\displaystyle\int_C(y-z)\,\mathrm ds=-\sqrt{145}\int_0^{2\pi}t\,\mathrm dt=\boxed{-2\pi^2\sqrt{145}}[/tex]

The sum of two numbers is 9.9, and the sum of the squares of the numbers is 53.21. What are the numbers?

Answers

Answer:

There are two possibilities:

[tex]x_{1} = 3.5[/tex] and [tex]y_{1} = 6.4[/tex]

[tex]x_{2} = 6.4[/tex] and [tex]y_{2} = 3.5[/tex]

Step-by-step explanation:

Mathematically speaking, the statement is equivalent to this 2-variable non-linear system:

[tex]x + y = 9.9[/tex]

[tex]x^{2} + y^{2} = 53.21[/tex]

First, [tex]x[/tex] is cleared in the first equation:

[tex]x = 9.9 - y[/tex]

Now, the variable is substituted in the second one:

[tex](9.9-y)^{2} + y^{2} = 53.21[/tex]

And some algebra is done in order to simplify the expression:

[tex]98.01-19.8\cdot y +2\cdot y^{2} = 53.21[/tex]

[tex]2\cdot y^{2} -19.8\cdot y +44.8 = 0[/tex]

Roots are found by means of the General Equation for Second-Order Polynomials:

[tex]y_{1} \approx \frac{32}{5}[/tex] and [tex]y_{2} \approx \frac{7}{2}[/tex]

There are two different values for [tex]x[/tex]:

[tex]y = y_{1}[/tex]

[tex]x_{1} = 9.9-6.4[/tex]

[tex]x_{1} = 3.5[/tex]

[tex]y = y_{2}[/tex]

[tex]x_{2} = 9.9 - 3.5[/tex]

[tex]x_{2} = 6.4[/tex]

There are two possibilities:

[tex]x_{1} = 3.5[/tex] and [tex]y_{1} = 6.4[/tex]

[tex]x_{2} = 6.4[/tex] and [tex]y_{2} = 3.5[/tex]

What’s the correct answer for this question?

Answers

Answer:

A.

Step-by-step explanation:

Circle with centre (0,2) will have the equation

(x)²+(y-2)² = r²

Finding r² by distance formula using P(-7,2)

r² = (-7-0)²+(2-2)²

r² = (-7)²

r² = 49

So substituting in the equation

x²+(y-2)² = 49

A recipe for 1 loaf of bread calls for 2 cups of flour, 12 tablespoons of water, and 1 teaspoon of salt. The recipe can be scaled up to make multiple loaves of bread. Complete the table that shows the quantities to use for multiple loaves of bread.

Answers

Answer:

For 2 loaves of bread, 4 cups of flour, 24 tablespoons of water, and 2 teaspoons of salt are required

For 4 loaves of bread,  8 cups of flour, 48 tablespoons of water, and 4 teaspoons of salt are required

Step-by-step explanation:

Given: A recipe for 1 loaf of bread calls for 2 cups of flour, 12 tablespoons of water, and 1 teaspoon of salt

To find: the missing terms in the box

Solution:

As for 1 loaf of bread, 2 cups of flour, 12 tablespoons of water, and 1 teaspoon of salt are required

So, for 2 loaves of bread, double the quantity.

So, for 2 loaves of bread, 2×2=4 cups of flour, 12×2=24 tablespoons of water, and 1×2=2 teaspoons of salt are required

For 4 loaves of bread, double the quantity used to make 2 loaves of bread.

For 4 loaves of bread,  4×2=8 cups of flour, 24×2=48 tablespoons of water, and 2×2=4 teaspoons of salt are required

See the image attached.

An old saying in golf is "You drive for show and you putt for dough." The point is that good putting is more important than long driving for shooting low scores and hence winning money. To see if this is the case, data from a random sample of 69 of the nearly 1000 players on the PGA Tour's world money list are examined. The average number of putts per hole and the player's total winnings for the previous season are recorded. A least-squares regression line was fitted to the data. The following results were obtained from statistical software. the p-value for the test in question 3 is 0.0087. p-value for the test in question 3 is 0.0087. a correct interpretation of this result is that:____________

Answers

Answer:

With this P-value, we have statistical evidence to support the claim that there is a relationship between the average number of putts per hole and the player's total winnings.

Step-by-step explanation:

In this case, the hypothesis test has the following hypothesis:

Null hypothesis: there is no relationship between the average number of putts per hole and the player's total winnings.

Alternative hypothesis: there is relationship between the average number of putts per hole and the player's total winnings.

Then, a P-value of 0.0087 is, without doubt, a strong evidence that the null hypothesis is false.

With this P-value, we have statistical evidence to support the claim that there is a relationship between the average number of putts per hole and the player's total winnings.

What is the length of the line?
square root of 119
square root of 60
13
square root of 17

Answers

Answer:

  13

Step-by-step explanation:

The line is the hypotenuse of a right triangle that is 5 units high and 12 units wide. The Pythagorean theorem, or the distance formula, tells you the length is ...

  length = √(5² +12²) = √(25+144) = √169

  length = 13

A sports-equipment factory produces sports balls. On Monday, they produced 1,080 balls altogether. All of the balls they produced were either volleyballs or soccer balls. They produced 7 times as many volleyballs as soccer balls. How many soccer balls did the factory produce on Monday?

Answers

Answer: 135 soccer balls

Step-by-step explanation:

In this equation, let x represent the number of soccer balls, and 7x represent the number of volley balls.

7x + x = 1,080

8x = 1,080

Now divide 1,080 by 8 to get the number of soccer balls.

1,080 / 8 = 135

In case you’re wondering how to find the number of volley balls, just multiply 135 by 7

135 * 7 = 945

Suppose that you are holding your toy submarine under the water. You release it and it begins to ascend. The graph models the depth of the submarine as a function of time. What is the domain and range of the function in the graph?

Answers

Answer:

Domain :  0 ≤ t ≤ 3

Range :   -4 ≤ d ≤ 0

Step-by-step explanation:

The graph attached models the depth of submarine as a function of time.

Points on x-axis represent the time and points on y-axis represent increase in height of the submarine.

Domain of a function is represented by the points on x-axis.

Therefore, Domain :  0 ≤ t ≤ 3

Range of function is represented by te points on y-axis.

Therefore, Range :   -4 ≤ d ≤ 0

5x + 6y = 32


5x – 6y = 8


Which variable or variables will be eliminated when you add the system of equations?

Answers

Answer:

The variable y is eliminated.

Step-by-step explanation:

To add a system of equations, we add the common terms. x with x, y with y and values with values.

In this question:

5x + 6y = 32

5x – 6y = 8

Adding:

5x + 5x + 6y - 6y = 32 + 8

10x = 40

So the variable y is eliminated.

Answer:

10x 40

Step-by-step explanation:

What’s the correct answer for this?

Answers

Answer:

36

Step-by-step explanation:

Diameter AB bisects the chord

MO = NO

5x+3=6x

3 = 6x-5x

x = 3

Now

MN = 5x+3+6x

MN = 5(3)+3+6(3)

MN = 15+3+18

MN = 36

what’s the correct answer for this?

Answers

Answer:

A.

Step-by-step explanation:

98%

The probability of not winning the raffle is 100% - 2% = 98%

Trey is estimating the length of a room in his house. The actual length of the room is 17 m. Trey's estimate is 15 m.
Find the absolute error and the percent error of Trey's estimate. If necessary, round your answers to the nearest tenth.

Answers

Answer:

Absolute error = 2m

Step-by-step explanation:

Absolute Error = Measured Value - Actual Value

17m - 15m = 2m

Answer:

2m and 11.8%

Step-by-step explanation:

Absolute error is defined mathematically as;

Error/original value

[Original value - estimated value]

[17-15 ] = 2m

The percentage error would be

[Original value - estimated value]/ original value

[17-15 ] /17 × 100%

0.1176 × 100% = 11.76%

11.8%[ to the nearest tenth]

given:f(x)=x-7 and h(x)=2x+3 write the rule for f(h(x))

Answers

Answer:

Step-by-step explanation:

h(x) = 2x + 3

f(2x+3)

2x+3 - 7

2x - 4

You deposit $5000 each year into an account earning 4% interest compounded annually. How much will you have in the account in 35 years?

Answers

Answer:

$ 402,722.01

Step-by-step explanation:

Penni Precisely buys 100 worth of stock in each of three companies: Alabama Almonds, Boston Beans, and California Cauliflower. After one year, AA was up 20%, BB was down 25%, and CC was unchanged. For the second year, AA was down 20% from the previous year, BB was up 25% from the previous year, and CC was unchanged. If A, B, and C are the final values of the stock, then:______A) A = B = CB) A = B < CC) C < B = AD) A < B < CE) B < A < C

Answers

Answer:

E) B < A < C

Step-by-step explanation:

Penni Precisely buys 100 worth of stock in each of three companies: Alabama Almonds, Boston Beans, and California Cauliflower.

Initial Values are: A, B and C

After one year, A was up 20%, B was down 25%, and C was unchanged.

Values after one year:

A=(100%+20%) of $100=$120 worth of stock.B=(100-25%) of $100=$75 worth of stock.C=100% of $100=$100 worth of stock.

For the second year, A was down 20% from the previous year, B was up 25% from the previous year, and C was unchanged.

Values after the second year

A=(100%-20%) of $120 =80% of $120=$96 worth of stock.B=(100%+25%) of $75 =125% of $75=$93.75 worth of stock.C=100% of $100 =$100 worth of stock.

Therefore, from the final values of the stock, we have that:

B<A<C

The correct option is E.

Find the largest prime factor of 309^2 - 147^2.

Answers

Answer: 2^4*3^5*19

Step-by-step explanation:

You also need to divided by 2 four times, then divided by 3 five times, then the final one is divided by 19.

The largest prime factor of 309² - 147² is, 19.

What is Greatest common factors?

The highest number that divides exactly into two more numbers, is called  Greatest common factors.

Given that;

The expression is,

⇒ 309² - 147²

Now, We can simplify as;

⇒ 309² - 147²

⇒ (309 - 147) (309 + 147)

⇒ 162 × 456

⇒ 2 × 3 × 3 × 3 × 3 × 2 × 2 × 2 × 3 × 19

Thus, The largest prime factor of 309² - 147² is,

⇒ 19

Learn more about the Greatest common factors visit:

https://brainly.com/question/219464

#SPJ2

An excavation crew is digging a tunnel under a bay. The crew has dug 573 meters of the tunnel, which is 34 meters past the halfway point. What will be the length of the tunnel when the crew has finished digging?

Answers

Answer:

The total length of the tunnel when the crew has finished digging is 1078 meters

Step-by-step explanation:

total length of the tunnel dug by the crew =  573 meters

let the halfway point of the tunnel = h

if the crew digs 34 meters past the halfway point, then we will this equation below;

h + 34 meters = 573 meters

h = 573 - 34

h = 539 meters

halfway point of the tunnel is 539 meters

Then, the total length of the tunnel when the crew has finished digging = 2h

= 2 x 539 meters

= 1078 meters

Therefore, the total length of the tunnel when the crew has finished digging is 1078 meters

Courtney sold 48 cantaloupes at the farmers' market and had 25 left. Which equation could be used to find x, the number of cantaloupes Courtney had originally?

Answers

Answer:

48+25=x

Step-by-step explanation:

Answer:

The answer is D :)

Thank me later.

OMG THIS QUESTION IS SO HARD WILL RATE IF U GET IT

Answers

Answer:

290

Step-by-step explanation:

(11*8)*2=176, (11*3)*2=66, (3*8)*2=48, 48+176+66= 290

PLEASE HELP AS QUICKLY AS POSSIBLE THANK YOU :)​

Answers

Answer:

C.

Step-by-step explanation:

Some rectangles can be squares, since they fit the criteria for both if it is a square, but not ALL rectangles can be squares, since squares have ALL 4 sides congruent, whereas rectangles only have 2 sets of sides congruent.

Other Questions
what is the process of making larger molecules by removing water called Which equation is not a linear function? The art teacher needs 70 markers for her classes. The markers come in packages of 15. How many packages of markers will the art teacher need to buy? ASAP! GIVING BRAINLIEST! Please read the question THEN answer CORRECTLY! NO guessing. I say no guessing because people usually guess on my questions. Compound sentences are incomplete if they are missing a ______ 60 POINTS!!!!!!!!!!!!!! yes really 60 POINTS Please Help The number of wins for a high school football team is measured for the season. When the team plays at home, it is generally believed that they will win. Compathe location of the game and the number of wins, a correlation coefficient of 0.891 is calculated. What would this imply about the football team winning at homeThe scatter plot would closely resemble a straight line with a positive slope. The data has a strong, positive correlation, but causation cannot bedeterminedThe scatter plot would closely resemble a straight line with a positive slope. The data has a strong positive correlation, and a causal relationship existsbetween the team playing at home and winning.The scatter plot would not be represented by a line of best fit with a positive slope. There is a weak correlation between the football team playing at homeand winningThere is no causation and almost no correlation between the football team playing at home and winning A circle representing a pool is graphed with a center at the origin. Grant enters the pool at point A and swims over to a friend who is located at point B. A coordinate plane with a circle drawn. The center of the circle is the origin, (0, 0). The circle has a diameter of 16 units. There is a line segment drawn from Point A at (8, 0), passing through (0, 4) to Point B at (negative 4, 6). Which equation represents Grants path? y = 2 4x y = 4 y = 6 y = 8 2x Which of the following technological advancements made the events described in the passage possible for the Ottomans? Armor Cannon Compass Harness Not sure which one.... If banks are prevented from engaging in risky practices, predatory lending is criminalized, trading in derivatives is made transparent and backed up by capital, and Wall Street lobbyists are held in check, then accountability and responsibility will be restored to the financial system, "too big to fail" will become a thing of the past, and a repeat of the Great Recession will be avoided. pls help thx love yaa. Explanationb. Illustrationc. Conditional statementd. Warning renes Dairy is deciding whether or not to enter the market for ice cream, currently monopolized by Matties Ice-cream. If it enters the market, Matties can either accommodate him and share his 10million in profits equally with Irene or fight him and cause a 5million loss for each in a price war. What would the Nash equilibrium be in this sequential game? Given the directrix of y = 6 and focus of (0, 4), which is the equation of the parabola? y = one fourthx2 + 5y = one fourthx2 5 y = one fourthx2 + 5 y = one fourthx2 5 What are the answers to a b and c ? given that a*b=2a-3b,then 2*(-3)= Find the value of 4 + (27 - 12 2) 2. the perimeter of a pickleball court is 128 feet. The width is represented by x and the length is represented by 2x + 4. What are the actual dimensions of the pickleball court (length and width)? Marcia's wages are 260 per week.She wants to save some money to buy a computer.The computer costs 218.Marcia is going to save 5% of her wages each week.How many weeks will it take her to save enough money to buy the computer? The product of two consecutive positive integers is 132. What are the integers? explain in full sentencesWhat is mental abuse? consider an electric dipole lying along x-axis with mid-point O as the origin of coordinate system.find the electric potential V at point P due to dipole.in addition to this,find electric potentials for special cases. when P lies on the axial line and when P lies on the equatorial line.